Sample Questions for Logical Reasoning (Part 2)

Sample Question for logical reasoning

In a group of five, each person has exclusive different preferences for soft drinks, smartphone brands, and bike companies. Soft drink preferences are Pepsi, Sprite, Limca, Coca Cola, and Thumbs up. Bike company preferences are Suzuki, Kawasaki, Honda, TVS, and Hero. Smartphone preferences are Nokia, Samsung, Apple, Micromax, and Motorola.

Bunty likes TVS and Pepsi but does not prefer Samsung or Apple. The one who likes Kawasaki prefers Apple. Mayuri likes Hero, Thumbs up and Motorola. Anil likes Limca and Nokia. Vandana prefers Suzuki and Sprite. Hemant does not prefer Samsung.

Which of the following combinations is correct?

  1. Bunty – Pepsi – TVS – Apple
  2. Vandana – Sprite – Suzuki – Samsung
  3. Hemant – Coca cola – Honda – Apple
  4. Anil – Limca- Kawasaki – Nokia

Answer. (2) From the given information, we can make the following table:

PersonSoft DrinkBikeSmartphone
BuntyPepsiTVSMicromax
MayuriThumbs UpHeroMotorola
AnilLimcaHondaNokia
VandanaSpriteSuzukiSamsung
HemantCoca ColaKawasakiApple

Hence, the combination Vandana – Sprite – Suzuki – Samsung is correct.

Read the information given below and then answer the questions that follow:

A lawyer must schedule appointments with eight clients F, G, H, I, J, K, L, and M in one week, Monday to Friday. She must schedule two appointments for Monday, Tuesday, and Wednesday, and one each for Thursday and Friday.

She must see H on Thursday.

She must see G before the day on which she sees I.

She must see J before the day on which she sees L.

She must see F before the day on which she sees L.

She must see K and F on the same day.

Question 1. Which of the following is an acceptable schedule for the week’s appointments?

 MondayTuesdayWednesdayThursdayFriday
a)G, MI, LK, FHJ
b)G, MI, JK, FHL
c)G, IM, LJHK, F
d)L, GK. MK, MHF

Answer. (b) Since the question does not supply any additional information, we just apply each condition to the options. The first condition states that there are two appointments for Monday, Tuesday, Wednesday, and one appointment on the other two days. So, we eliminate (c) because it not possible to schedule two appointments on Friday.

The second condition states that H must be seen on Thursday, and our remaining choices reflect that. Then, the next condition states that G must come on a day before which I have his appointment, and our remaining choices reflect that this is right. Fourth condition states that F should come on a day before which L has his appointment. So, (a) and (d) are eliminated as well.

This leaves us with (b) as the correct choice.

Question 2. Which of the following cannot be true?

  1. She sees M on Monday.
  2. She sees K on Tuesday.
  3. She sees L on Thursday.
  4. She sees I on Wednesday.

Answer. (c) Option (a) is not likely to be correct because M is not under any specific restriction. So, M could go on any day. As for (b), though K is under a certain restriction, the import of that restriction is that K should come earlier in the week. Therefore, (b) is probably not the correct choice. Look at (c), L is under the restriction that he should be scheduled later than both J, F, and L. Due to this, L should be scheduled in the later part of the week. This choice, however, has L early in the week.

The way to determine whether it is possible for L to have an appointment on Tuesday is to assume that he can. Now we should see whether the assumption is consistent with the initial condition or not. If L is scheduled for Tuesday, then J and F both must be scheduled for Monday. But that it is not possible as K would also have to be scheduled for Monday. As for (d), you can construct schedules and show that it is possible.

Question 3. If the lawyer sees I on Tuesday, then which of the following must be true?

  1. She sees J on Monday.
  2. She sees M om Tuesday.
  3. She sees K on Tuesday.
  4. She sees L on Friday.

Answer. (d) Start by entering additional information in a table:

MondayTuesdayWednesdayThursdayFriday
I  H 

G must come before I:

MondayTuesdayWednesdayThursdayFriday
GI H 

K and F must schedule for the same day, which must now be Wednesday

MondayTuesdayWednesdayThursdayFriday
GIK, FH 

L must be later in the week than F, however:

MondayTuesdayWednesdayThursdayFriday
GIK, FH 

So, J and M must have appointments on Monday and Thursday, though not necessarily in that order. The tables show that (a) and (b) are possible true. But, (c) and (d) are false.

Statements:

  • All books are notebooks.
  • No notebook is pen.

Inferences:

  •  No book is pen.
  •  All pens are books.
  1. Only inference (A) follows.
  2. Only inference (B) follows.
  3. Both (A) and (B) follow.
  4. Neither (A) nor (B) follows.

Answer. (a) All books are notebooks and no notebook is pen. Hence, (a) follows.

  • Statements:
  •  All scientists working in America are intelligent.
  •  Some Indian scientists are working in America.

Inferences:

  •  All intelligent scientists are Indian.
  •  None of the Indian scientists are intelligent.
  •  Some Indian scientists are intelligent.
  •  Some intelligent Indian scientists have migrated to America.
  1. Both inference (A) and (B) follow.
  2. Both inference (C) and (D) follow.
  3. Inference (A), (C), and (D) follow.
  4. None of the inference follows.

Answer. (b) From the statements, it can be concluded that some Indian scientists are working in America, and all scientists working in India are intelligent. Hence inference (C) follows.

Further, it is also implied in statements that some Indian scientists have migrated to America. Hence inference (D) also follows.